ChaseDream
搜索
返回列表 发新帖
查看: 4695|回复: 2
打印 上一主题 下一主题

lsat 21-2-1

[复制链接]
楼主
发表于 2004-12-17 22:15:00 | 显示全部楼层

lsat 21-2-1

7. A neighborhood group plans to protest the closing of the neighborhood's only recreation center on the grounds that to do so would leave the neighborhood without local access to a recreation center "Our neighborhood already has the most residents per center of any neighborhood in the city" complained one resident, "and closing this center would make the situation unacceptable since access to recreational facilities is a necessity for this neighborhood"


Each of the following if true weakens the resident's argument EXCEPT (A) A large number of the neighborhood's residents are unable to travel outside their locality to gain access to recreational facilities (B) Children, the main users of recreational facilities make up a disproportionately small segment of the neighborhood's population (C) Often the recreation center in the neighborhood is open but not being used. (D) Programs that are routinely filled at other recreation centers must be canceled at the neighborhood's recreation center due to lack of interest (E) As people become more involved in computers and computer games recreation centers are becoming increasingly less important


我选了e ,答案是a。 我认为a 的确weaken了阿, e 说重要性降低了但是还是有影响啊。

沙发
 楼主| 发表于 2004-12-21 19:33:00 | 显示全部楼层
ding
板凳
 楼主| 发表于 2004-12-23 09:37:00 | 显示全部楼层

总是犯这个错误

谢谢斑竹

您需要登录后才可以回帖 登录 | 立即注册

Mark一下! 看一下! 顶楼主! 感谢分享! 快速回复:

所属分类: 法学院申请

近期活动

正在浏览此版块的会员 ()

手机版|ChaseDream|GMT+8, 2024-6-16 08:43
京公网安备11010202008513号 京ICP证101109号 京ICP备12012021号

ChaseDream 论坛

© 2003-2023 ChaseDream.com. All Rights Reserved.

返回顶部